Download Osteoarthritis Pre Post Test - 44 KB

Survey
yes no Was this document useful for you?
   Thank you for your participation!

* Your assessment is very important for improving the workof artificial intelligence, which forms the content of this project

Document related concepts
no text concepts found
Transcript
Osteoarthritis Pre/Post Test
Osteoarthritis Pre/Post Test
1. A 65 year old woman with an 18 year history of rheumatoid arthritis is evaluated for
increasingly severe right groin pain of 6 months' duration. Her pain awakens her at night and
causes significant difficulty in walking. On initial diagnosis of her rheumatoid arthritis, she was
rheumatoid factor and anticyclic citrullinated peptide antibody positive. For the past 5 years,
her only medications have been methotrexate and infliximab, which have significantly
alleviated her inflammation and improved her function and limited the progression of visible
joint damage in her hands and wrists.
On physical examination, temperature is 37 °C (98.6 °F). She has an obvious right leg limp.
There are moderately severe rheumatoid deformities of the wrist, metacarpophalangeal, and
proximal interphalangeal joints. On musculoskeletal examination, range of motion of the right
hip elicits pain and flexion is limited to 85 degrees. Internal rotation of the right hip also is
markedly limited and painful.
Laboratory Studies
Hemoglobin 11.5 g/dL (115 g/L)
Leukocyte count 8,700/μL (8.7 x 10⁹ /L)
Platelet count 350,000/μL (350 x 10⁹ /L)
Erythrocyte sedimentation rate 25 mm/h
Creactive protein 1.5 mg/dL (15 mg/L)
Aspiration of the right hip joint guided by ultrasonography yields 1 cc of clear fluid with a
leukocyte count of 1200/μL (1.2 x 10⁹ /L) (60% mononuclear cells)
Which of the following is the most likely cause of this patient's hip pain?
A. Osteonecrosis
B. Hip fracture
C. Secondary osteoarthritis
D. Septic arthritis
Osteoarthritis Pre/Post Test
2. A 67 year old man is evaluated for a 3 year history of right knee pain exacerbated by
ambulation. Fulldose nonsteroidal anti-inflammatory drug therapy and acetaminophen, 4
g/d, does not sufficiently relieve his symptoms. He is overweight, and weight loss and exercise
have been recommended previously however, he has not been compliant with these
recommendations.
Musculoskeletal examination reveals bilateral knee crepitus and a right knee effusion that is
slightly warm to the touch. However, there is no significant tenderness of the joint capsule.
Radiographs of the right knee show moderate medial jointspace narrowing with subchondral
sclerosis and tibial osteophytes. Arthrocentesis of the right knee yields 30 mL of yellow fluid.
Synovial fluid leukocyte count is 800/μL (0.8 x 10⁹ /L), and no crystals are observed.
Which of the following is the most appropriate treatment for this patient?
A. Short course of prednisone
B. Intraarticular corticosteroids
C. Arthroscopic lavage
D. Topical nonsteroidal anti-inflammatory drugs
Osteoarthritis Pre/Post Test
3. A 62 year old man with a history of coronary artery disease is evaluated for the sudden onset
of melena. He has no abdominal pain and is hemodynamically stable. Six weeks ago, he
discontinued therapy with acetaminophen and switched to ibuprofen therapy to treat
recalcitrant chronic left hip pain attributed to osteoarthritis. One year ago, he had a myocardial
infarction.
Upper esophagogastroduodenoscopy reveals a small gastric ulcer but shows no active bleeding.
Endoscopic biopsy specimens are negative for Helicobacter pylori. A proton pump inhibitor is
begun.
Which of the following is the most appropriate treatment for this patient's osteoarthritis?
A. Lowdose ibuprofen
B. Intraarticular hyaluronan
C. Intraarticular corticosteroids
D. Tramadol
4. A 62 year old obese man is evaluated for a 1 year history of progressively worsening right
knee pain. Six weeks ago, he developed bleeding due to endoscopyproven nonsteroidal antiinflammatory drugrelated gastric ulcer. He also has hypertension. Medications are atenolol,
enalapril, and omeprazole. Musculoskeletal examination reveals bilateral knee crepitus on
range of motion. There is a 4 degree valgus deviation of the right knee.
Which of the following is the most appropriate management for this patient?
A. Weight loss; quadriceps strength training; acetaminophen
B. Diclofenac
C. Indomethacin suppositories
D. Arthroscopic debridement
Osteoarthritis Pre/Post Test
5. A 65 year old woman is evaluated for a 2 year history of joint pain and stiffness of the hands.
The appearance of her hands on physical examination is shown (Image of arthritic hand). Which
of the following is the most appropriate treatment for this patient?
A. Acetaminophen
B. Methotrexate
C. Prednisolone
D. Etanercept
ANSWERS
Osteoarthritis Pre/Post Test
Osteoarthritis Pre/Post Test
1. A 65 year old woman with an 18 year history of rheumatoid arthritis is evaluated for
increasingly severe right groin pain of 6 months' duration. Her pain awakens her at night and
causes significant difficulty in walking. On initial diagnosis of her rheumatoid arthritis, she was
rheumatoid factor and anticyclic citrullinated peptide antibody positive. For the past 5 years,
her only medications have been methotrexate and infliximab, which have significantly
alleviated her inflammation and improved her function and limited the progression of visible
joint damage in her hands and wrists.
On physical examination, temperature is 37 °C (98.6 °F). She has an obvious right leg limp.
There are moderately severe rheumatoid deformities of the wrist, metacarpophalangeal, and
proximal interphalangeal joints. On musculoskeletal examination, range of motion of the right
hip elicits pain and flexion is limited to 85 degrees. Internal rotation of the right hip also is
markedly limited and painful.
Laboratory Studies
Hemoglobin 11.5 g/dL (115 g/L)
Leukocyte count 8,700/μL (8.7 x 10⁹ /L)
Platelet count 350,000/μL (350 x 10⁹ /L)
Erythrocyte sedimentation rate 25 mm/h
Creactive protein 1.5 mg/dL (15 mg/L)
Aspiration of the right hip joint guided by ultrasonography yields 1 cc of clear fluid with a
leukocyte count of 1200/μL (1.2 x 10⁹ /L) (60% mononuclear cells)
Which of the following is the most likely cause of this patient's hip pain?
A. Osteonecrosis
B. Hip fracture
C. Secondary osteoarthritis
D. Septic arthritis
Osteoarthritis Pre/Post Test
2. A 67 year old man is evaluated for a 3 year history of right knee pain exacerbated by
ambulation. Full dose nonsteroidal anti-inflammatory drug therapy and acetaminophen, 4 g/d,
does not sufficiently relieve his symptoms. He is overweight, and weight loss and exercise have
been recommended previously however, he has not been compliant with these
recommendations.
Musculoskeletal examination reveals bilateral knee crepitus and a right knee effusion that is
slightly warm to the touch. However, there is no significant tenderness of the joint capsule.
Radiographs of the right knee show moderate medial jointspace narrowing with subchondral
sclerosis and tibial osteophytes. Arthrocentesis of the right knee yields 30mL of yellow fluid.
Synovial fluid leukocyte count is 800/μL (0.8 x 10⁹ /L), and no crystals are observed.
Which of the following is the most appropriate treatment for this patient?
A. Short course of prednisone
B. Intraarticular corticosteroids
C. Arthroscopic lavage
D. Topical nonsteroidal anti-inflammatory drugs
Osteoarthritis Pre/Post Test
3. A 65 year old man with a history of coronary artery disease is evaluated for the sudden onset
of melena. He has no abdominal pain and is hemodynamically stable. Six weeks ago, he
discontinued therapy with acetaminophen and switched to ibuprofen therapy to treat
recalcitrant chronic left hip pain attributed to osteoarthritis. One year ago, he had a myocardial
infarction. Upper esophagogastroduodenoscopy reveals a small gastric ulcer but shows no
active bleeding. Endoscopic biopsy specimens are negative for Helicobacter pylori. A proton
pump inhibitor is begun.
Which of the following is the most appropriate treatment for this patient's osteoarthritis?
A. Lowdose ibuprofen
B. Intraarticular hyaluronan
C. Intraarticular corticosteroids
D. Tramadol
4. A 65 year old obese man is evaluated for a 1 year history of progressively worsening right
knee pain. Six weeks ago, he developed bleeding due to endoscopyproven nonsteroidal antiinflammatory drugrelated gastric ulcer. He also has hypertension. Medications are atenolol,
enalapril, and omeprazole. Musculoskeletal examination reveals bilateral knee crepitus on
range of motion. There is a 4 degree valgus deviation of the right knee.
Which of the following is the most appropriate management for this patient?
A. Weight loss; quadriceps strength training; acetaminophen
B. Diclofenac
C. Indomethacin suppositories
D. Arthroscopic debridement
Osteoarthritis Pre/Post Test
5. A 65 year old woman is evaluated for a 2 year history of joint pain and stiffness of the
hands. The appearance of her hands on physical examination is shown (Image of arthritic
hands). Which of the following is the most appropriate treatment for this patient?
A. Acetaminophen
B. Methotrexate
C. Prednisolone
D. Etanercept